Five students are lined up in a row. How
many arrangements could be made if
the position of the last boy remains
unchanged?
(WAEC)​

Answers

Answer 1

Step-by-step explanation:

16 arrangement can be done

Answer 2

The 24 arrangements could be made if the position of the last boy remains unchanged.

Arrangement

Arrangement is a plans or preparations for a future event.

How to solve this problem?

The steps are as follow:

Given, Five students are lined up in a rowWe have make the arrangment such that last boy should remain unchangedTo find how many arrangements are possible in a set of objects, use the formula below, where x is the number of objects.

x! , where,! is factorial

x! is equal to x*(x-1)*(x-2)*(x-3)*…(x-(x-1))

In this case we have to take x equal to 4 because last boy to remain unchanged

∴ 4*3*2*1 = 24 arrangments

Therefore total 24 arrangements could be made if the position of the last boy remains unchanged.

Learn more about arrangment here:

https://brainly.com/question/251701

#SPJ2


Related Questions

Find the greatest number such that; x360y is divisible by 24
WILL GIVE BRAINIEST, NEED ANSWER ASAP

Answers

Answer:

93600

Step-by-step explanation:

24=3*2*2*2

y=0,8

1) x3600, then x=9,6,3

2)x3608, then x=1,4,7

9 is the largest so the final answer is 93600

On Monday, a localamburger shop said a combined total of 225 hamburgers and cheeseburgers. The number of cheeseburgers sold was two times the number
of hamburgers sold. How many hamburgers were sold on Monday?
hamburgers
х
?

Answers

Answer:

h = hamburgers sold

2h - 72 = 578

2h = 650

2       2

h = 325

There were 72 fewer cheeseburgers sold than hamburgers. That means cheeseburgers = 325 - 72

325 - 72 =  253

253 = cheeseburgers

325 = hamburgers

253 + 325 = 578

Hamburgers = 325

Hope this helps :)

Find the constant of proportionality if y is
proportional to x.

A. 25
B. 21
C. 26
D. 28

HELPPP!!!!!!

Answers

B. 21 because 21+21=42 42+21=63 etc

If f(x) = 2x2 + 1, what is f(x) when x = 3?

Answers

Answer:

18

Step-by-step explanation:

f(x)=2x(2+1)

f(3)=2(3)(2+1)

    =6(2+1)

    =12+6

    =18

In a group of 26 pupils, 3 play the flute only.
5 play the piano only.
7 play neither instrument.
How many pupils play both instruments?

Answers

26 minus 3 is 23 , 23 minus 5 is 18, 18 minus 7 is 11 so i’m pretty sure


answer =11

Here is a Venn diagram:

Solve the system of equations.
5x – 4y = -10
y = 2 x – 5
x =
y =

Answers

Answer:

6x2+3y2=12,x+y=2,2x-y=-1x+y=4,x-y=2

Step-by-step explanation:

I just know.

Answer:

Step-by-step explanation:

5x – 4y = -10

y = 2 x – 5

Rearrage the equation:

5x – 4y = -10 (1)

-2 x +y =  5.  (2)

pluging variable y equation2 in 1

5x - 4•(2x-5) = -10

 -3x = -30

Solve equation [1] for the variable  x  

3x = 30

x = 10

x = 10

y = 2x-5

Use the  x  value to solve for  y  

y = 2(10)-5 = 15

x= 10

y= 15

The length of a rectangle is 7 ft less than three times the width, and the area of the rectangle is . Find the dimensions of the rectangle.

Answers

Answer:

See explanation

Step-by-step explanation:

the length of a rectangle is 7 feet less than three times it’s width. if the area of the rectangle is 180 ft, find the dimensions of the rectangle.

Width = x

Length = 3x - 7

Area of a rectangle = length × width

180 = (3x - 7) * x

180 = 3x² - 7x

3x² - 7x - 180 = 0

Using quadratic formula

a = 3

b = -7

c = -180

x = -b ± √b² - 4ac / 2a

x = -(-7) ± √(-7)² - 4(3)(-180) / 2(3)

x = 7 ± √49 - (-2160) / 6

= 7 ± √49 + 2160 / 6

x = 7 ± √2209/6

= 7 ± 47 / 6

= (7 + 47)/6 or ( 7 - 47)/6

= 54/6 or -40/6

x = 9 or -6.67

x cannot be negative

Therefore,

x = 9 ft

Width = x = 9 ft

Length = 3x - 7

=3(9) - 7

= 27 - 7

= 20 ft

If 4/3 . sin42⁰ = x then 4/3 . cos48⁰ = ?​

Answers

Step-by-step explanation:

If 4/3 . sin42⁰ = x then 4/3 . cos48⁰ = ?

so it's that

Rewrite the equation 8x + 6x2 - 7= in standard form and identify a, b, and c.
Select one:
a. a =5,b= 8.c = -7
b.a=6,6= 8,0 = 1
ca=5,b=8,0= 7
da= 8,b= 6.c = -7
40

Answers

Answer:

a = 6

b = 8

c = -7

Step-by-step explanation:

In standard form, we have ;

y = ax^2 + bx + c

Here, we have;

6x^2 + 8x - 7

a is the coefficient of x^2 which is 6 in this case

b is the coefficient of x which is 8

c is the last number which is -7

So we have;

a = 6

b = 8

c = -7

Tariq wants to make some purple paint.
He mixes blue, red and white paint in the ratio 5:3:2
Tariq needs 60 litres of purple paint.
He gets a discounted price of 60% on the red paint.
Calculate how many litres of red paint he buys at the discounted price.

Answers

Answer:

18 liters

Step-by-step explanation:

Blue : red : White = 5 : 3 : 2

Quantity of Blue paint = 5x

Quantity of Red paint = 3x

Quantity of White paint = 2x

Total paint = 5x + 3x + 2x = 10x

10x = 60 liters

x = 60/10

x = 6

Quantity of red paint = 3x = 3*6 = 18

3 years later a mother will be 4 times as
old as her son. 3 years ago ,the mother age was two times as old as her son age will be 8 years hence. What are their present ages?​

Answers

Answer:

33 and 6 years

Step-by-step explanation:

Mother's age = mSon's age = s

3 years later:

m + 3 = 4(s + 3) ⇒ m = 4s + 12 - 3 = 4s + 9

3 years ago:

m - 3 = 2(s + 8) ⇒ m = 2s + 18 + 3 = 2s + 21

Solve for s by substitution:

4s + 9 = 2s + 212s = 12s = 6

Find the value of m:

m = 4*6 + 9 = 33

Mother is 33 and son is 6

A corporate team-building event cost $4, plus an additional $3 per attendee. If there are 39 attendees, how much will the corporate team-building cost?

Answers

Answer:

$121

Step-by-step explanation:

Find how much additional money it will cost from the attendees:

39(3)

= 117

Add the other $4:

117 + 4

= 121

So, it will cost $121


Find angle m A. 69
B. 23
C. 46
D. 167

Answers

Answer:

a

Step-by-step explanation:

∠DAF - ∠CFA=∠DFC

 118-49 =69

What is the answer to this question

Answers

26 m rhiendjfnrbsjen

Answer:

26 m is the answer to the question

Someone help me please I’m exhausted & struggling

Answers

Answer:

Step-by-step explanation:

What is the simplified value of the expression below?
1/3 divided by 2/3

0

1/3

1/2

1

Answers

1/2
Is the answer
To the problem

Answer:

option C : 1/2

Step-by-step explanation:

[tex]\frac{1}{3} \div \frac{2}{3} \\\\\frac{\frac{1}{3}}{\frac{2}{3}}\\\\\frac{1}{3} \times \frac{3}{2} \\\\\frac{1}{2}[/tex]

maths questions on coordinate geomery

Answers

Answer:

:

Example Question #1:

Step-by-step explanation:

Which of the following is the equation of a line that is parallel to the line 4x – y = 22 and passes through the origin?

Possible Answers:

4x – y = 0

(1/4)x + y = 0

4x + 8y = 0

4x = 8y

y – 4x = 22

Correct answer:

4x – y = 0

Explanation:

We start by rearranging the equation into the form y = mx + b (where m is the slope and b is the y intercept); y = 4x – 22

Now we know the slope is 4 and so the equation we are looking for must have the m = 4 because the lines are parallel. We are also told that the equation must pass through the origin; this means that b = 0.

In 4x – y = 0 we can rearrange to get y = 4x. This fulfills both requirements.

', .

What is the slope of the line? What is the y-intercept of the line? y = 2x + 5

Answers

Slope intercept form of a line is, y = mx + c where m is the slope and c is constant.

Judging the given equation y = 2x + 5

Slope (m) of the line is 2,

y-intercept of the line,

y = 2x + 5

y = 2×0 + 5

y = 5

Answered by GAUTHMATH

Answer:

m = 2

y intercept = 5

Step-by-step explanation:

The given equation of the line is ,

[tex]\implies y = 2x +5[/tex]

We know that the Standard equation of Slope Intercept Form of the line is,

[tex]\implies y = mx + c[/tex]

Where ,

m is slope c is y intercept

On comparing to the Standard form of the line we get ,

[tex]\implies Slope = 2 [/tex]

[tex]\implies y - intercept= 5[/tex]

what is the value of x? (3x-14)°=180° [4(x-9)]°=180°

Answers

Answer:

3x-14=180

3x=194

x= 64 2/3

4(x-9)=180

4x-36=180

4x=216

x=54

Hope This Helps!!!

Step-by-step explanation:

(3x-14)°=180°

3x-14=180

3x=180+14

3x=194

x=64.6

[4(x-9)]°=180°

4x-36=180

4x=180+36

4x=216

x=54

I need help with this, Im never good with these

Answers

Answer:

(a) basketball

(b) hat

(c) stuffed animal

explanation:

I'm not sure if got right but from what i see;

Chances from 1-23 you get a stuffed boi

Chances from 24-54 you get a hat (why)

Chances from 55-100 you get a basketball

Winner 5 got 90, which is between 55-100

Winner 6 got 35, which is between 24-54

winner 8 got a 7, which is between 1-23

Hope I didn't read it wrong

Which graph shows the solution to the system of linear inequalities?

x – 4y < 4

y < x + 1

On a coordinate plane, 2 straight lines are shown. The first solid line has a positive slope and goes through (0, negative 1) and (4, 0). Everything above the line is shaded. The second dashed line has a positive slope and goes through (negative 1, 0) and (0, 1). Everything below the line is shaded.

On a coordinate plane, 2 straight lines are shown. The first dashed line has a positive slope and goes through (negative 1, 0) and (0, 1). Everything above of the line is shaded. The second solid line has a positive slope and goes through (0, negative 1) and (4, 0). Everything above the line is shaded.

On a coordinate plane, 2 straight lines are shown. The first solid line has a positive slope and goes through (0, negative 1) and (4, 0). Everything above the line is shaded. The second solid line has a positive slope and goes through (negative 1, 0) and (0, 1). Everything below the line is shaded.

On a coordinate plane, 2 straight lines are shown. The first solid line has a positive slope and goes through (0, negative 1) and (4, 0). Everything below the line is shaded. The second dashed line has a positive slope and goes through (negative 1, 0) and (0, 1). Everything below the line is shaded.

Answers

Answer:

its the 2nd

Step-by-step explanation:

The graph showing common region to both inequalities represents the solution coordinates that satisfy both inequalities.

What is inequality in mathematics?

In mathematics, an inequality is a relation which makes a non-equal comparison between two numbers or other mathematical expressions.

Given are the following inequalities -

x – 4y < 4

y < x + 1

We have the following inequalities -

x – 4y < 4

y < x + 1

Inequality [1]

x - 4y < 4

x - 4 < 4y

4y > x - 4

y > x/4 - 1

Inequality [2]

y < x + 1

Refer to the graph attached. The common region to both inequalities represents the solution.

Therefore, the graph showing common region to both inequalities represents the solution coordinates that satisfy both inequalities.

To solve more questions on inequalities, visit the link below-

brainly.com/question/26434651

#SPJ2

which of the following functions is graphed below?

Answers

Answer:

d number on is the wright answer

hope it may help you

What is the y-intercept of the line y = -3x + 7?
O A. -7
O B. 3
O C. 3
O D. 7

Answers

Answer:

Answer would be

D: 7

It’s D which is positive 7

A fruit punch mixture is made of Orange juice, Pineapple juice, and Cranberry juice in a 3:5:7 ratio. How much of each juice type do you need for a 375 ml mixture?

Answers

Answer: a good amount

Step-by-step explanation:

because if you just get a table spoon for all flavour mixtures then wala they are all the same amount -__-------------i is not smart at all srry

You would get a mixture of a really good smoothie Thankyou for the idea

common factors of 8xy⁴,6x²y,10x²y²​

Answers

Answer:

[tex]2^{3}[/tex]×5×3×[tex]x^{2}[/tex]×[tex]y^{4}[/tex]

Step-by-step explanation:

[tex]8xy^{4}[/tex]= [tex]2^{3}[/tex]×[tex]x[/tex]×[tex]y^{4}[/tex]

6[tex]x^{2} y[/tex]= 2×3×[tex]x^{2}[/tex]×[tex]y^{}[/tex]

10[tex]x^{2} y^{2}[/tex] = 2×5×[tex]x^{2}[/tex]×[tex]y^{2}[/tex]

[tex]2^{3}[/tex]×5×3×[tex]x^{2}[/tex]×[tex]y^{4}[/tex]

Regan earns $1380 per month out of which he saves $1140 per month. What is the ratio (in the lowest form) of Regan's income to his savings? $$

Answers

Answer:

23:19

Step-by-step explanation:

This questions is looking for this ratio:

income:savings

Regan's income is $1,380 per month.

Regan's savings is $1,140 per month.

Substitue those numbers in our original ratio, and you'll get this:

1380:1140

However, this question wants the lowest form of the ratio.

To do that, you can find the GCF (greatest common factor) and divide both number's by it, or you can just randomly divide them by small numbers you know both of them are divisible by.

I'm going to use the 2nd method because it's usually quicker for big numbers like these:

1380/10 = 138

1140/10 = 114

138/2 = 69

114/2 = 57

69/3 = 29

57/3 = 19

29 and 19 are prime numbers, so you can't divide them anymore. This is how we know we've reached the lowest terms of these numbers.

So the answer is 29:19

Hope this helps (●'◡'●)

Answer: 23:19

Step-by-step explanation: 1380 and 1140 LCM is 60. 1380/6 = 23 and 1140/6= 19

Therefore, the answer is 23:19

Quadrilateral A B C D is shown. All sides are congruent. Angle C is a right angle.
Which statements are true regarding quadrilateral ABCD? Check all that apply.

ABCD has congruent diagonals.
ABCD is a rhombus.
ABCD is not a rectangle.
ABCD is not a parallelogram.
ABCD has four congruent angles.

Answers

Answer:

The first one and last one

Step-by-step explanation:

Answer:

1,2,5

Step-by-step explanation:

got it right on edge

A triangle has the vertices of P(2,5),Q(-4,7) and R(-2,-1) classify the triangle is scalene, Isoceles or an equilateral triangle

Answers

Answer:

isoceles

Step-by-step explanation:

The vertices are not equal

In the equation 17x2 = 12x, the value of c is:
O
0 12
O 17

Answers

Answer:

ok ok ok ok ok ok ok

Step-by-step explanation:

GIVING 15 POINTS PLS HELP!!!

Part A: factor 2x^2b^2 + 5xb^2 + 2b^2. Show your work.
Part B: factor x^2 - 8x + 16. Show your work.
Part C: factor x^2 - 49. Show your work.

Answers

Answer:

the picture I've given is an answer

Other Questions
help me with this problem please!!!! what is the approximate value of x in the diagram below? Which item is an example of a secondary source? Suppose you invest a certain amount of money in account that earns 3% annual interest. You also invest that same amount + $2000 that earns 4% annual interest. If the total interest from both accounts at the end of the year is $535, how much has been invested in each account? Swifty Corporation purchased a truck at the beginning of 2020 for $109600. The truck is estimated to have a salvage value of $4100 and a useful life of 123000 miles. It was driven 18000 miles in 2020 and 26000 miles in 2021. What is the depreciation expense for 2020?a. $37752 b. $22308 c. $16639 d. $15444 Judy, a 28 y/o, presents to the clinic with a fever, vaginal discharge, and pain in the lower abdomen, pelvis, and lower back. These symptoms are accompanied by chills, nausea, and vomiting. This presentation is most typical of: In a right triangle, the lengths of the two legs are 8 cm and 10 cm respectively. Find the hypotenuse of the triangle.9 cm10.5 cm12 cm12.8 cm According to Okun's law, if the unemployment rate goes from 5% to 3%, what will be the effect on the GDP?A. It will increase by 7%.B. It will decrease by 7%.C. It will decrease by 1%.D. It will increase by 1%. amy shoots a 100 arrows at a target each arrow hits with a probability 0.01 what is the probability that one of her first 5 arrows hit the target Amy, a high-strung teenager, was suddenly startled by a loud bang that sounded like a gunshot. Her heartbeat accelerated rapidly. When she realized that the noise was only a car backfiring, she felt greatly relieved but her heart kept beating heavily for several minutes more. What hormone was released into her bloodstream that is causing this response While some property owners choose to perform both the property and asset manager functions themselves, many commercial property owners choose to employ professional property managers instead. The property manager works under a management contract in which the manager is empowered to serve as the owner's fiduciary. This type of relationship is more commonly referred to as a(n) Suppose a life insurance company sells a $240,000 one-year term life insurance policy to a 19-year-old female for $240. The probability that the female survives the year is 0.999578. Compute and interpret the expected value of this policy to the insurance company. The expected value is $ (Round two decimal places as needed.) Which statement best describes the effect of the alliteration in this excerpt? A 80C la presion de vapor del benceno (C6H6) es de 1 atm. Calcula la cantidad de hexano (C6H14) que debemos aadir a 200g de benceno para que su presion de vapor sea de 700mm de Hg CAN SOME HELP ME PLEASE AND THANK YOU ANSWER CORRECTLY OR ILL REPORT YOU How can political instability make genocide more likely?by causing hardships and famine for the general populationby encouraging those who disagree to live together in a part of the cityby creating an unequal distribution of power between groupsby causing courts to punish members of certain groups In the introduction of the Letter of Credit example, to where did the ficticious Florentine wool merchant travel to buy fine wool? The management of a diverse group of customers with different needs within the same service setting is called: Which of the following statements considered as always true? A. All intersecting lines are perpendicular. B. All parallel lines cut by a transversal line. C. All perpendicular lines are intersecting line. D. All transversal line is for parallel lines only What is the highest peak in North America? A. Mt. Saint Helen B. Mt. McKinley C. Mt. Whitney D. Mt. Rainier Please select the best answer from the choices provided A B C D